Sie sind auf Seite 1von 31

Series

Objectives: At the end of the module students are able to


Identify the dierence between sequence and series
Identify the properties of some special series such as geometric series, harmonic
series and pseries
Determine whether a series converges or diverges using test of convergence
Evaluate the radius and interval of convergence of a series

Innite Series

Innite series is formed by adding terms of an innite sequence:


Sequence: {an } = a1 , a2 , a3 , , an ,
n=1
Series:

n=1

an = a1 + a2 + a3 + + an +

Denition 1.1:
The kth partial sum Sk of the series is the sum of rst k terms:
k

Sk =
n=1

an = a1 + a2 + + ak

For example, the partial sum of rst few terms are


S1 = a1
S2 = a1 + a2
S3 = a1 + a2 + a3
S100 = sum of rst 100 terms of an
Example:
(i) Sequence:
Series:

n=1

1
1 1 1
= , , ,
n
2
2 4 8
1
1 1 1
= + + +
n
2
2 4 8
1

(ii) Sequence:
Series:

k=0

1
1 1
(1)k+1
= 1 , , , ,
k+1
2
3 4
k+1
(1)
1
1
1
= 1 + +
+ +
k+1
2
3
4

Denition 1.2:

th
Given a sequence
partial sum Sn is the sum of
k=1 ak = {a1 , a2 , a3 , }, the n
rst n terms of the sequence, i.e., Sn = a1 + a2 + a3 + + an = n ak . The partial
k=1
sum of rst n terms are

S1 = a1
S2 = a1 + a2
S3 = a1 + a2 + a3
.
.
.
Sn = a1 + a2 + a3 + + an
The column on the left (in red) forms a sequence
{Sk }n = S1 , S2 , S3 , , Sn
k=1
If the sequence of the partial sums {Sn } converges to L, then the sum of the series
converges to L, i.e.,
lim Sn = L

S =

an = L

n=1

where L is a nite number. The limit L is the sum of the innite series
may write it as
L=

n=1

an and we

an = a1 + a2 + a3 + a4 +

If the sequence of partial sum {Sn } is not convergent, then it is said to be divergent.
Example 1.1:
Find the limit of the innite series

n=1

1
2n

Solution:
We write down the rst four partial sum of the series
1
2
3
1 1
=
+ =
2 4
4
1 1 1
7
=
+ + =
2 4 8
8
1 1 1
1
15
=
+ + +
=
2 4 8 16
16

S1 =
S2
S3
S4

and from the pattern of the values we conclude that the partial sum is
Sn =

1
2n 1
=1 n
n
2
2

So that the limit


lim Sn = lim

The innite sum of the series


S =

n=1

1
2n

=1

1
=1
2n

converges to 1.
Example 1.2:
Find the limit of the innite series

n=1

1
n(n + 1)

Solution:
We write down the rst four partial sum of the series
1
2
1 1
2
=
+ =
2 6
3
1 1
1
3
=
+ +
=
2 6 12
4
1 1
1
1
4
=
+ +
+
=
2 6 12 20
5

S1 =
S2
S3
S4

and from the pattern of the values we conclude that the partial sum is
n
n+1

Sn =
3

The innite sum of the series


1
n
= lim
n 1 +
n n + 1

S = lim Sn = lim
n

1
n

=1

converges to 1.
Example 1.3:
Determine whether the series with the sequence
1 , 1 , 1 , 1 , 1 , 1 ,
is convergent/divergent.
Solution:
S1 = 1 =

1 1
+
2 2

1 1

2 2
1 1
= 1 + (1) + 1 = 1 = +
2 2

S2 = 1 + (1) = 0 =
S3

S4 = 1 + (1) + 1 + (1) = 0 =

1 1

2 2

and from the pattern of the values we conclude that the partial sum is
Sn =

1 1
+ (1)n+1
2 2

We may write
Sn =

1
0

if n is odd
if n is even

Since {Sn } oscillates between 1 and 0, it follows that limn Sn does not exist, hence
the series diverges.

Geometric Series

Geometric Series is a series of numbers having a common ratio r such that


n

Sn =
k=1

ar k1 = a + ar + ar 2 + ar 3 + + ar n2 + ar n1

where a is the rst term and r is the ratio. We note that the nth term is an = ar n1 .
As we multiply the series with r, we have
n

r Sn =

k=1

ar k = ar + ar 2 + ar 3 + ar 4 + + ar n1 + ar n

Then we subtract the rst series with the second series and nd that
r Sn Sn = ar n a
(r 1)Sn = a(r n 1)
a(r n 1)
Sn =
r1

Then, the innite sum of the geometric series is


lim Sn =

lim

lim

a(r n 1)
r1
ar n
a

r1 r1

If the ratio |r| < 1, the rst term on the right hand side vanishes (please refer to Theorem
3 in Sequence),
a
S =
1r
So that the geometric series converges if |r| < 1 and diverges if |r| > 1.
Example 2.1:
Determine whether the series converges/diverges,
5

10 20 40
+

+
3
9
27

Solution:
The series is also equivalent to
5 + (5)

2
2
+ (5)
3
3

+ (5)

2
3

Therefore a = 5 and r = 2 . Since |r| = 2 < 1, then the series is convergent and the
3
3
sum of the innite series is
a
5
S=
=
=3
1r
1 ( 2 )
3
Example 2.2:
Determine whether the series conveges/diverges,
1 + 0.4 + 0.16 + 0.064 +
5

Solution:
The series can be rewritten as
1 + 0.4 + (0.4)2 + (0.4)3 +
Therefore, a = 1 and r = 0.4. Since |r| < 1, the series is convergent and the sum of the
innite series is
1
5
S=
= = 1.66
1 0.4
3
Example 2.3:
Write the number
2.317 = 2.31717171717...
as a fraction (ratio of integer).
Solution:
The number can be rewritten as
=
=
=
=

2.31717171717...
2.3 + 0.017 + 0.00017 + 0.0000017 +
17
17
17
2.3 +
+
+
+
1000 100000 100000
17
17
17
2.3 +
+
+
+
3
5
(10)
(10)
(10)7
17
17
1
17
1
+

+
2.3 +
3
3 (10)2
3 (10)4
(10)
(10)
(10)

a + a r + a r2 +

From the second term onward are of the form of Geometric Series with a =
r=

1
. Thus,
(10)2
2.317 = 2.3 +
= 2.3 +

a
1r

17
(10)3
1
(10)2

1
23
17
1147
=
+
=
10 990
495

Example 2.4:
Is the series

22n 31n

n=1

17
and
(10)3

convergent/divergent?
Solution:

22n 31n =

n=1

n=1

n=1

(22 )n 3 3n
4n 3

1
3n

4n
=
3 n
3
n=1
= 3

4
3

n=1

The innite sum is a Geometric Series with a =


diverges.

4
3

and r = 4 . Since |r| > 1, the series


3

Exercise:
1.) Show that the series
2+

2
2
2
+ 2 ++ n +
3 3
3

converges and nd its sum.


2.) Determine whether the series converges, if so, nd the innite sum.
i)
ii)

1 1 1
+ 1+
8 4 2

(3)n1
4n
n=1

Telescoping Series

Telescoping Series is a series whose sum appears to have the situation that almost every
term cancels with either a preceding or succeeding term. For example, the series
S =

n=1

1
=
n(n + 1)

n=1

1
1

n n+1

we consider the partial sums of this series up to kth term


k

1
1

n n+1

Sk =
n=1

1
2

1 1
1 1
1

2 3
3 4
4
1
1
1
1
1
+
+

k1
k1 k
k k+1

++

We can see that the terms with the same colour are canceled with each other. Thus, the
terms which survive are the rst and the last number:
Sk = 1

1
k+1

Therefore, the sum of the innite series is


S = lim Sk = lim
k

1
k+1

=1

The series converges to 1.

Harmonic Series

The harmonic series, given by

n=1

1
1 1 1
= 1+ + + +
n
2 3 4

is divergent innite series in spite of the fact that the limit of the sequence when n goes
to innity is zero. The proof of the divergence of the harmonic series is shown below by
comparing the harmonic series with another divergent series:
1 1 1 1 1 1 1
+ + + + + + +
2 3 4 5 6 7 8
1 1 1 1 1 1 1
> 1+ + + + + + + +
2 4 4 8 8 8 8
1+

1
2

1
2

where each colour term of the harmonic series is greater than the corresponding term
in the second series with the same colour, and therefore the sum of the harmonic series

must be greater than the sum of the second series. However, the sum of the second
series is innite,
1+
= 1+

1
2
1
2

+
+

1 1
1 1 1 1
+
+
+ + +
4 4
8 8 8 8
1
1
+
+ =
2
2

It follows that the innite sum of the harmonic series must be divergent.
Example 4.1:
Show that each of the following series are divergent.
(i)

n=1

3
n

(ii)

n=3

1
n

Solution:
(i) By the fact that constant can be factored out of the series as follows,

n=1

3
1
=3
n
n
n=1

1
Since n is divergent, any constant multiplies a divergent series it will still be
n=1
divergent and so the series has to be divergent.

(ii) The rst two terms of the harmonic series is

n=1

1
1
1
= 1+ +
n
2 n=3 n
=

n=3

1
=
n

3
1
+
2 n=3 n

n=1

1 3

n 2

As we subtract a constant from a divergent series, the series will still be divergent.
In order word, an innity minus a constant which is still innity. So that the series
is divergent.

Determining Convergence/Divergence Of Series

5.1
If lim an = 0 then the series
n

nth Term Test

an is divergent.

n=1

If lim an = 0, further investigation is necessary to determine whether the series


n

an is convergent or divergent. In other word, it is not guarantee that the series

n=1

converges.
Example 5.1:
Determine whether the series

n=1

n2
5n2 + 4

is convergent/divergent.
Solution:
By using nth term test,
n2
1
1
= lim
=0
4 =
2+4
n 5n
n 5 + 2
5
n
lim

thus the series diverges.


Example 5.2:
Determine the following series, by using nth term test, whether they are convergent/divergent?
1. Given the series:

n=1

By using nth term test:

n
2n + 1

n
1
= =0
n 2n + 1
2
lim

Conclusion: Diverges
2. Given the series:

n=1

10

1
n2

By using nth term test:

1
=0
n n2
Conclusion: Further investigation is needed.
lim

3. Given the series:

n=1

By using nth term test:

1
lim = 0
n
n
Conclusion: Further investigation is needed.

4. Given the series:

n=1

en
n

By using nth term test:


en
en
= lim
=
n n
n 1
lim

Conclusion: Diverges
Remark:
If limn an = 0, then the series may either converges or diverges. We need to use other
test techniques to conrm.
Theorem 1:
Given a series

n=1

an = a1 + a2 + a3 +

For any positive integer k, if the sum of the same series is taken from (k + 1)th term
onwards

n=k+1

an = ak+1 + ak+2 + ak+3 +

then both series are either convergent or divergent.


Example 5.3:
Show whether the following series

n=1

1
1
1
1
=
+
+
+
(n + 2)(n + 3)
34 45 56
11

convergent/divergent.
Solution:
Recall the series we have considered earlier (Telescoping series):

n=1

1
1
1
1
1
1
=
+
+
+
+
+
n(n + 1)
12 23 34 45 56

1
1
1
=
+
+
1 2 2 3 n=1 (n + 2)(n + 3)
The series in the question is the sum of this series started from third term onwards, this
means that

1
1
=
(n + 2)(n + 3) n=3 n(n + 1)
n=1
1
As we found before the series n(n+1) converges to 1, and thus by the above Theorem,
n=1
the series in the question is convergent and converges to

S =

n=1

1
1
1
1
=1

=
(n + 2)(n + 3)
12 23
3

Theorem 2:
If

an and

bn are convergent series with sums A and B, respectively, then

i)

(an + bn ) converges with sum A + B

ii)

(c an ) converges with sum c A, for every c = 0

iii)

(an bn ) converges with sum A B

Example 5.4:
Find the sum of series

n=1

3
1
+ n
n(n + 1) 2

Solution:
By using Theorem 2,

n=1

3
1
+ n
n(n + 1) 2

=3

n=1

1
1
+
n(n + 1) n=1 2n

From the previous results where the rst series is a telescoping series and the second
series has been considered in Example 1.1, the sum of the series given in the question is
12

convergent and converges to 3 1 + 1 = 4


Example 5.5:
Find the sum of series

n=1

1
1
+
n
5
n

Solution:
By using the theorem,

1
1
+
5n n

n=1

n=1

1
1
+
5n n=1 n

Since the rst series is a geometric series with r = 1 < 1, this series is convergent. The
5
second series is a harmonic series, it is divergent. The sum of these two series is thus
divergent.

5.2

pSeries

The pseries is a series of the following form

n=1

1
1
1
1
1
= p + p + p + p +
np
1
2
3
4

Theorem 3:
Given a pseries,

n=1

1
np

If p > 1, the series is convergent.


If p 1, the series is divergent.
e.g.,
The series

n=1

1
1
1
1
1
= 4 + 4 + 4 + 4 +
4
n
1
2
3
4

is convergent because it is a pseries with p = 4 > 1.


The series

n=1

1
n1/2

1
11/2

1
21/2

1
31/2

is divergent because it is a pseries with p = 1/2 < 1.


13

1
41/2

Exercise:
Determine whether the following series is convergent or divergent:
i) 1 +
ii)

1
1
1
1
+
+
+
+
8 27 64 125
n1.4 + 3n1.2

n=1

iii)

n=1

(n + 1)2
n(n + 2)

5.3

Comparison Test

Assume 0 an bn for all n.


Suppose that the sum of series
bn is convergent, then the series
an is also
convergent. In other word, a series of positive terms is convergent if its terms
are less than the corresponding terms of a positive series which is known to be
convergent.
Suppose that the sum of series
an is divergent, then the series
bn is also
divergent. In other word, a series is divergent if its terms are greater than the
corresponding terms of a series which is known to be divergent.
Example 5.6:
Determine whether the series

n=1

2n2

5
+ 4n + 3

converges/diverges.
Solution:
Since n 1, thus 2n2 + 4n + 3 > 2n2 , then

5
5
< 2
2n2 + 4n + 3
2n

The series

n=1

5
5
=
2
2n
2

n=1

1
n2

is a pseries with p = 2 > 1, by Theorem 3 the series is convergent. Therefore, by


Comparison Test the series

n=1

2n2

5
+ 4n + 3
14

is convergent .

Example 5.7:
Determine whether

n=3

ln n
n

converges/diverges.
Solution:
Since ln(e) = ln(2.7183) = 1, therefore, ln n > 1 for n 3. Thus,

1
ln n
>
for n 3 .
n
n
1
We know that the series n is a harmonic series and it is divergent. By Theorem 1
n=1
1
the series n=3 n is also divergent, and by Comparison Test,

n=3

ln n
n

diverges .

Exercise:
Determine, by using Comparison Test, whether the series converges or diverges:
i)

n=1

2
n4 + 2

ii)

n=1

5.4

4 + 3n
2n

Limit Comparison Test

Given the positive terms series

an

and

n=1

bn

n=1

We require that all an and bn are positive. Suppose


an
lim
=c
n bn
where c > 0 is a nite number.
If the limit of

an
bn

is positive,
an
= c > 0,
n bn
lim

then the sums an and bn are either both convergent or both divergent.
n=1
n=1
In other word, if the limit is positive, the terms of these two series are growing at
the same rate, so either both series converge or diverge together.
15

n
If the limit of an is zero, and the sum bn converges, then the sum an converges.
b
In other word, if the limit is zero, the bottom terms are growing more quickly than
the upper terms. So that if the bottom series converges, the upper series which is
growing more slowly must also converge.
n
If the limit of an is innite, and the sum bn diverges, then the sum an diverges.
b
In other word, if the limit is innite, the bottom series is growing more slowly. If
the bottom series diverges, the upper series must also diverge.

Remark:
The Limit Comparison Test only applies to series with non-negative terms, i.e., an , bn >
0 for all n.
Example 5.8:
By using Limit Comparison Test, determine whether the series

n=1

2n

1
1

converges/diverges.
Solution:
Let
an =

2n

We choose
bn =
Then,
an
lim
=
n bn
Since the series

n=1

1
2n 1
1
2n

1
1

1
2n

1
=1>0
n 1 1
2n

= lim

1
is a convergent geometric series (please see Example 1.1), by
2n

Limit Comparison Test, the series

n=1

2n

1
converges.
1

Example 5.9:
By using Limit Comparison Test, determine whether the series

k=1

2k 2 + 3k

5 + k5

16

converges/diverges.
Solution:
The highest power of the numerator is 2k 2 and the highest power of the denominator
proportional to k 5/2 . We let
2k 2 + 3k
ak =
5 + k5
and choose
2k 2
2
bk = = 1/2
k
k5
Then the limit
an
lim
=
k bn

2k 2 +3k

5
lim 5+k
2
k
k 1/2
2

2k + 3k k 1/2
lim

k
2
5 + k5
3
2
k (2 + k )
k 1/2
= lim

k 5/2
2
5
k
+1
k5

Since
n=1 bk is pseries with p =

2k 2 + 3k

diverges.
5 + k5
k=1

2+

lim

1
,
2

5
k5

3
k

=1>0

+1

it is divergent. By Limit Comparison Test,

Exercise:
By Limit Comparison Test, determine whether the following series converges or diverges.
i)

k=1

4k 2 2k + 6
8k 7 + k 8

ii)

n=1

17

3n

5
+1

5.5
Given a series

The Ratio Test

an .

n=1

an+1
i) If lim
= L < 1, then the series
n
an
an+1
ii) If lim
= L > 1, then the series
n
an
an+1
iii) If lim
= L = 1, then the series
n
an

an is convergent.

n=1

an is divergent.

n=1

an may be convergent or convergent.

n=1

Example 5.10:
Use the ratio test, determine whether the given series is convergent/divergent:
i)

n=1

1
n!

ii)

k=1

(2k)!
4k

iii)

k=1

1
2k 1

Solution:
i) Let an =

1
.
n!

By ratio test,
an+1
lim
n=
an

= lim

n=

1
(n+1)!
1
n!

n!
n= (n + 1)!
1
= lim
=0<1
n= n + 1
= lim

Therefore, the series is convergent.


ii) Let ak =

(2k)!
.
4k

By ratio test,
ak+1
lim
k=
ak

2(k+1) !
4k+1
(2k)!
4k

= lim

k=

(2k + 2)! 1

k=
(2k)!
4
1
(2k)!(2k + 1)(2k + 2)
=
lim
4 k=
(2k)!
1
=
lim (2k + 1)(2k + 2) = +
4 k=
= lim

18

Therefore, the series is divergent.


iii) Let ak =

1
.
2k1

By ratio test,
ak+1
lim
k=
ak

= lim

k=

1
2(k+1)1
1
2k1

2k 1
k= 2k + 1
1
2 k
= lim
=1
k= 2 + 1
k

= lim

Therefore, the series may be converges or diverges.

Exercise:
Use ratio test to determine whether the following series are convergent/divergent.
i)

k=1

3k
k!

ii)

n=1

5.6
Consider the sum

1
5n

The Root Test

an .

n=1

1
n

1. If lim |an | = L < 1, then the series


n

1
n

2. If lim |an | = L > 1, then the series


n

an converges absolutely.

n=1

an diverges.

n=1

3. If lim |an | n = 1, then we may conclude nothing from this (inconclusive).


n

Example 5.11:
Use the Root Test to determine whether the following series
i)

k=1

4k 5
2k + 1

ii)

n=1

converge or diverge.

19

1
ln(n + 1)

Solution:
i) By using Root Test,
lim

4k 5
2k + 1

1
k

4k 5
k 2k + 1
5
4 k
= lim
=2>1
k 2 + 1
k
=

lim

Therefore, the series diverges.


ii) By using Root Test,
lim

1
ln(n + 1)

1
n

n
1
= lim
n
ln(n + 1)
1
= lim
=0<1
n ln(n + 1)

Therefore, the series converges.

Exercise:
Determine, by using Root Test, whether the following series
1)

n=1

2)

k=1

3n + 1
2n 1
k

1e

3)

k=1

4)

nn
2 (16n )
1

nn

k=1

converges/diverges.

5.7

The Alternating Series Test (AST)

An alternating series is a series whose terms are alternately positive and negative, e.g.,

n=1

(1)n1
1 1 1 1 1
= 1 + + +
n
2 3 4 5 6

20

The alternating series test is used when the terms of the underlying sequence alternate.
Suppose that we have a series

n=1

(1)n1 an = a1 a2 + a3 a4 +

where an 0 for all n N. Then if


i) {an } is a decreasing sequence, i.e., a1 a2 a3 , and
ii) lim an = 0,
n

then the series is convergent.


Example 5.12:
Use AST determine whether the following series
1)

n=1

(1)n1
n

2)

k=1

(1)k+1 (k + 3)
k(k + 1)

converges/diverges.
Solution:
1) The series is given by

n=1

(1)n1
1 1 1 1 1
= 1 + + +
n
2 3 4 5 6

i) we observe that an > an+1 because


ii) limn an = limn

1
n

1
n

>

1
,
n+1

and

= 0.

Thus, both conditions of AST are satised and it may conclude that the series is
convergent.
2)

i) To show ak > ak+1 ,


(k + 3)
k+4

k(k + 1) (k + 1)(k + 2)
(k + 2)(k + 3) k(k + 4)
=
k(k + 1)(k + 2)
2
k + 5k + 6 k 2 4k
=
k(k + 1)(k + 2)
k+6
=
>0
k(k + 1)(k + 2)

ak ak+1 =

21

for all k 1. We found that ak ak+1 > 0 ak > ak+1 which satises the rst
condition of AST.
ii) The limit
3
1+ k
k+3
lim
= lim
=0
k k(k + 1)
k k + 1

Thus, the second condition of AST is satised and the series converges.

Exercise:
Use AST determine whether the following series
1)

n=1

(1)n+1
n2

2)

(1)k+1 ek

k=1

converges/diverges.

Types of Convergence

A series
an is called absolutely convergent if the series of absolute value of
convergent. E.g., the series

n=1

|an | is

(1)n1
1
1
= 1 2 + 2
2
n
2
3

is absolutely convergent because

n=1

(1)n1
=
n2

n=1

1
1
1
= 1+ 2 + 2 +
n2
2
3

is convergent because it is a pseries with p = 2 > 1. If


is divergent, we call the series conditionally convergent.

an is convergent and

Theorem 4:
If a series
an is called absolutely convergent, then the series is convergent.
Example 6.1:
The series

n=1

(1)n1
1 1
= 1 +
n
2 3
22

|an |

is convergent (please see Example 5.12), however the series

(1)n1
=
n

n=1

n=1

1
1 1
= 1+ + +
n
2 3

is divergent (because it is Harmonic series). Therefore, the series is conditionally convergent.

6.1

The Ratio Test for Absolute Convergence

Given an alternating series

n=1

an .

an+1
= L < 1, then the series
an
therefore converges).

n=1

If lim

If lim

an+1
= L > 1, then the series
an

an+1
= 1, then the series
n
an
(inconclusive).

n=1

If lim

n=1

an is absolutely convergent (and

an diverges.

an may either be convergent or divergent

Example 6.2:
Determine the type of convergence of the following series

(1)n

n=1

n3
3n

Solution:
Let
an = (1)n

n3
,
3n

then an+1 = (1)n+1

(n + 1)3
3n+1

By using ratio test


lim

an+1
an

(1)n+1 (n + 1)3 /3n+1


n
(1)n n3 /3n
(n + 1)3 3n
= lim
3
n
3n+2
n
3
1 n+1
= lim
n 3
n
=

lim

1
= lim
n 3

1+
1

1
n

Therefore, by ratio test, the series converges absolutely.

23

1
3

Power Series

Objective: At the end of the module, you should be able to


Identify properties of the power series,
determine radius of convergence,
determine interval of convergence, represent some functions using power series.

7.1

What is Power Series?

It is a series of the form

n=0

cn xn = c0 + c1 x + c2 x2 + c3 x3 +

where x is variable and cn s are constant coecient of the series.


E.g.,
i)

n=0

ii)

n=0

xn = 1 + x + x2 + x3 +
x3 x5
(1)n x2n+1
=x
+

2n + 1
3
5

For each xed x value, the series becomes as what we had learned earlier (no variable) and
we can use the appropriate techniques to test whether the series converges or diverges.
E.g.,

n=0

xn = 1 + x + x2 + x3 +

If x = 2, the series becomes

n=0

2n = 1 + 2 + 22 + 23 +

which is divergent.
1
If x = , the series becomes
2

n=0

1
2

=1+

1
+
2

1
2

which is convergent (geometric series with r = 1/2).


24

1
2

What we need to do here is to nd what is/are the possible value of x that will make
the series converges. There is a number R such that the power series will converge for
|x| < R and will diverge for |x| > R. The number R is called the radius of convergence
of the series. This can be written in interval form as
|x| < R

R < x < R

This is called interval of convergence. Note that the series may or may not converge if
|x| = R. There are four possibilities of interval of convergence:
i) R < x < R ,

(R, R)

ii) R x < R ,

[R, R)

iii) R < x R ,

(R, R]

iv) R x R ,

[R, R]

Therefore, we must check whether the end point values should be included or not so
that the power series will converge for these values. To completely identify the interval
of convergence all that we have to do is determine if the power series will converge for
x = R and x = R.
E.g.,

n=0

xn = 1 + x + x2 + x3 +

This is a geometric series, we know for this series to converge when


|x| < 1 or

1 <x <1

Here, the radius of convergence R = 1. The end point values 1 and 1 are not inclusive.
Example 7.1:
For what values of x is the series

n!xn

n=0

converges?
Solution:
By Ratio Test,
lim

an+1
an

(n + 1)!xn+1
n
n!xn
n!(n + 1)xn x
= lim
n
n!xn
= lim |(n + 1)x|
=

lim

lim (n + 1)|x| =

25

The series diverges for all values of x except when x = 0, thus, the radius of convergence
R = 0.
Example 7.2:
Find the radius of convergence and the interval of convergence of the series

n=0

(1)n xn
n+1

Solution:
By Ratio Test,
lim

(1)n+1 xn+1 /(n + 2)


(1)n xn /(n + 1)

xn+1
n+1

n n + 2
xn
n+1
= lim |x|
n
n+2
1
1+ n
= lim |x|
2 = |x|
n
1+ n
=

lim

For the series to converge, |x| < 1. Thus, the radius of convergence R = 1, and the
interval of convergence is 1 < x < 1. To test the inequality signs let x = 1, then

n=0

(1)n (1)n
=
n+1

n=0

(1)2n
=
n+1

n=0

1
=
n+1

n=1

1
n

This is a Harmonic series and thus divergent. So that x = 1 is not included in the
interval of convergence. Now let x = 1,

n=0

(1)n (1)n
=
n+1

n=0

(1)n
=
n+1

n=1

(1)n1
n

By AST, the series converges (See Example 5.12). So that x = 1 is included in the
interval of convergence. Therefore the interval of convergence is 1 < x 1.
Example 7.3:
Find the radius of convergence and the interval of convergence of the series

n=1

(1)n1 xn
n3

26

Solution:
By Ratio Test,
(1)n xn+1 /(n + 1)3
lim
n
(1)n1 xn /n3

xn+1
n3
= lim
n
n (n + 1)3
x
=
=

n
n+1

lim

lim

1
1+

1
n

|x|
|x|

= |x|
The series converges if |x| < 1. Therefore, the radius of convergence R = 1. For nd the
interval of convergence, we consider rst at x = 1,

n=1

(1)n1 (1)n
=
n3

n=1

(1)2n1
1
=
3
n
n3
n=1

where 2n 1 is an odd number for all integer n and thus (1)2n1 = 1. Since the last
series is a pseries with p = 3, thus the series converges. At x = 1,

n=1

(1)n1 (1)n
=
n3

n=1

(1)n1
(1)n
=
n3
n3
n=1

By AST where
i) {1/n3 } is a decreasing sequence, and
ii) limn

1
n3

=0

the series converges. Therefore, the interval of convergence is 1 x 1.


Example 7.4:
Find the radius of convergence and the interval of convergence of the series

k=0

(1)k x2k
(2k)!

27

Solution:
By Ratio Test,
lim

ak+1
ak

(1)k+1 x2(k+1) /[2(k + 1)]!


k
x2k /(2k)!
(2k)!
x2k+2
2k
= lim
k (2k + 2)!
x
(2k)!
= lim
|x|2
k (2k)!(2k + 1)(2k + 2)
1
= lim
|x|2 = 0
k (2k + 1)(2k + 2)
=

lim

Thus, the series converges for all x and the radius of convergence R = and interval
of convergence is (, ).
Example 7.5:
Find the radius of convergence and the interval of convergence of the series
where
(2)n xn
an =
4
n

n=1

an

Solution:
By Ratio Test,

(2)n+1 xn+1 / 4 n + 1

= lim
n
(2)n xn / 4 n
n
= lim 2x 4
n
n+1

an+1
lim
n
an

lim 2x 4

1
1+

1
n

= | 2x| = 2|x|
Thus, the series converges when 2|x| < 1 or |x| < 1 . The radius of convergence R = 1 .
2
2
At x = 1 ,
2

(2)n ( 1 )n
(2)n (2)n
1
2

=
=
1
1
4
n
4
n4
n=1
n=1
n=1 n
Since this is a pseries with p =

n=1

1
4

1
1, it is divergent at x = 2 . When x = 1 ,
2

1
(2)n ( 2 )n

=
4
n

n=1

(2)n (2)n

=
4
n

By AST,
28

n=1

(1)n

4
n

i)

4
n

is a decreasing sequence, and

1
ii) lim = 0
4
n
n
the series converges by AST test. Therefore, the interval of convergence is 1 < x 1 .
2
2

7.2

Power Series centered at x = a

If a is a constant, and if x is replaced with x a, then the resulting series has the form

n=0

cn (x a)n = c0 + c1 (x a) + c2 (x a)2 + + cn (x a)n +

which is known as the power series centered at x = a or about x = a.


E.g.,
i)

n=0

ii)

k=0

(x 1) (x 1)2
(x 1)n
(x 1)n
=1+
+
++
+
n+1
2
3
n+1
(1)k (x + 3)k
(x + 3)2 (x + 3)3
= 1 (x + 3) +

+
k!
2!
3!

Theorem 5:
Given a power series
true:

n=0 cn (x

a)n , it is exactly one of the following statements is

i) The series converges only when x = a.


ii) The series converges for all x.
iii) There is a positive number R such that the series converges if |x a| < R and
diverges if |x a| > R.
iv) The series may converges or diverges at either of these values x = aR or x = a+R.
If the series converges for |x a| < R, then the value a is at the center between a R
and a + R:
Example 7.6:
Find the radius of convergence and the interval of convergence for the series

n=1

(x 5)n
n2
29

R
x
a

aR

a+R

Solution:
By Ratio Test,
lim

(x 5)n+1 /(n + 1)2


(x 5)n /n2

lim

(x 5)n+1
n2

(n + 1)2
(x 5)n
n
n+1

lim

1
1+

lim

1
n

|x 5|
|x 5|

= |x 5|
Thus, the series converges if |x 5| < 1. The radius of convergence is R = 1. The
interval is
|x 5| < 1

1 < x 5 < 1
51 <
x
<5+1
4<
x
<6

To check the end points:


When x = 6,

n=1

(x 5)n
=
n2

n=1

(6 5)n
=
n2

n=1

1
n2

This is a pseries with p = 2 > 1 and thus it is convergent.


When x = 4,

(4 5)n
(1)n
(x 5)n
=
=
n2
n2
n2
n=1
n=1
n=1
By AST,
i)

1
n2

is a decreasing sequence, and

1
= 0.
n n2

ii) lim

Thus, this series is convergent by AST. Therefore, the interval of convergence is 4 x


6.
30

Example 7.7:
Find the radius of convergence and the interval of convergence for the series

k=1

k(x + 2)k
3k+1

Solution:
By Ratio Test,
(k + 1)(x + 2)k+1 /3k+2
n
k(x + 2)k /3k+1
(k + 1)(x + 2)k+1 k(x + 2)k

= lim
n
3k+2
3k+1
k+1
x+2
= lim

n
k
3
x+2
=
3
lim

By Ratio Test, for the series to converge


x+2
< 1,
3

|x + 2| < 3

Therefore, the radius of divergence is R = 3. Now,


|x + 2| < 3

3 < x + 2 < 3
5 <
x
<1

To check the endpoints:


At x = 5,

k=1

k(x + 2)k
=
3k+1

k=1

k(3)k
=
3k 3

k=1

k(1)k 3k
1
=
k 3
3
3

(1)k k

k=1

This series is divergent.


At x = 1,

k=1

k(x + 2)k
=
3k+1

k=1

k(3)k
=
3k 3

k=1

k
3

This series is divergent. The interval of convergence is 5 < x < 1 or (5, 1).

End
31

Das könnte Ihnen auch gefallen